Search found 270 matches


Oops ... Did a calculation mistake

If Ron and Anurag and Stuart and Mike are sitting together.
The total group can be arrange = 4! x 2 x 2 = 96 (multiplied by 2 for each R A,A R and S M,M S arrangement)

So total - specific cases = 624

IMO: D

by neerajkumar1_1

Wed Feb 08, 2012 1:14 am
Forum: Problem Solving
Topic: the BTG experts
Replies: 16
Views: 2566

total number of cases 6! = 720 If Mike and Stuart are not sitting next to each other the number of case left will be 720 - 4! x 2 = 720 - 48 = 672 Now if Anurag and Ron also are not seated next to each other the number of cases to subtract will decrease further.. Hence total number of cases will inc...

by neerajkumar1_1

Wed Feb 08, 2012 1:05 am
Forum: Problem Solving
Topic: the BTG experts
Replies: 16
Views: 2566

If x is an integer, is x|x| < 2^x? 1) x < 0 2) x=-10 OA: D I chose E . Can someone explain why my reasoning below is incorrect? 1) Insufficient: Picked x = - 1 -1*|x| < 1/2 |x| > - 1/2 |x| = +/- 1 a. |x| = -1 => -1 < -1/2 => 1 > 1/2 => true b. |x| = 1 => 1 < -1/2 => false 2) Insufficient a. |x| = 1...

by neerajkumar1_1

Sat Feb 04, 2012 11:08 pm
Forum: Data Sufficiency
Topic: x|x|<2^x?
Replies: 4
Views: 1371

If x is an integer, is x|x| < 2^x? 1) x < 0 2) x=-10 OA: D I chose E . Can someone explain why my reasoning below is incorrect? 1) Insufficient: Picked x = - 1 -1*|x| < 1/2 |x| > - 1/2 |x| = +/- 1 a. |x| = -1 => -1 < -1/2 => 1 > 1/2 => true b. |x| = 1 => 1 < -1/2 => false 2) Insufficient a. |x| = 1...

by neerajkumar1_1

Sat Feb 04, 2012 11:07 pm
Forum: Data Sufficiency
Topic: x|x|<2^x?
Replies: 4
Views: 1371

hi all, request to assist me in solving the problem : i feel the answer is C. If the two horizontal lines in the figure provided are parallel, what is (a + f + d) - (e + h)? 1) a + f = 180 2) a - f = 60 a /b -------------------------------------------- c/d e f -----------------/--------------------...

by neerajkumar1_1

Sat Feb 04, 2012 10:59 pm
Forum: Data Sufficiency
Topic: data sufficiency -- parallel lines
Replies: 2
Views: 983
by neerajkumar1_1

Mon Jan 09, 2012 10:17 pm
Forum: Critical Reasoning
Topic: genetics
Replies: 6
Views: 1569

technologically advanced educational tools

In one study engineering students who prepared for an exam by using toothpicks and string did no worse than similar students who prepared by using an expensive computer with sophisticated graphics. In another study, military personnel who trained on a costly high-tech simulator performed no better o...

by neerajkumar1_1

Mon Jan 09, 2012 10:12 pm
Forum: Critical Reasoning
Topic: technologically advanced educational tools
Replies: 5
Views: 1715

I suppose mukgera u r highly confused... Please check the solution... I am replying to a post.. Please read carefully.. What I had posted earlier is to prove a scenario in which x is +ve and y is -ve and still we are getting x - y =1 and xy < 0 which according to ur post is not possible. what you co...

by neerajkumar1_1

Sun Oct 16, 2011 1:17 am
Forum: Data Sufficiency
Topic: *** Confused *** Is the product xy negative
Replies: 11
Views: 2101

exactly my point mukgera... i have not assumed x and y are integers.. pls read the question.. for any value of x... y will be negative in the given conditions and the product of xy will be -ve... we get a consistent answer... Hope this helps.. Hi neeraj, if we have x = 0.5 and y = -0.5 then x - y =1...

by neerajkumar1_1

Sun Oct 16, 2011 1:04 am
Forum: Data Sufficiency
Topic: *** Confused *** Is the product xy negative
Replies: 11
Views: 2101

Please can someone shed light on the answer for this DS question. Is the product xy negative? (1) x^2 – x < 0 (2) (x - 4)/(y- 3) = 1 The Manhattan explanation says the answer is (C). However, I think the answer is (B). Following is my reasoning. Statement (1): This statement does not tell us anyt...

by neerajkumar1_1

Sun Oct 16, 2011 12:13 am
Forum: Data Sufficiency
Topic: *** Confused *** Is the product xy negative
Replies: 11
Views: 2101

statement 1 m = 9s. which means that m is a multiple of 9 also m = 4n + 9 for this to be true, since we are already adding 9 to 4n, we will have to make 4n a multiple of 9 to satisfy m=9s hence n can take value 9, 18, so on so forth.. since n will also have to be a multiple of 9, there will be a con...

by neerajkumar1_1

Sun Oct 02, 2011 9:28 pm
Forum: Data Sufficiency
Topic: DS from MGMAT Adv Quant -- m=4n+9
Replies: 3
Views: 1323

hi ... just on a quick look.. . the first question... I would go for D... but i feel the question is not a gmat question... if the ratio is 5:9... and the first statement says The number of students who participates in either dance or choral program is prime, then in that case... there is only one s...

by neerajkumar1_1

Tue Aug 02, 2011 3:37 am
Forum: Data Sufficiency
Topic: Data Sufficiency : Some Good Questions
Replies: 7
Views: 1425

I still don't understand why E is correct here. though ur image is not completely visible... I can still guess the correct answer... the question is an inference based question... anything outside of what is written in the stimulus cannot be inferred... anything to do with the loudness of sound or ...

by neerajkumar1_1

Tue Aug 02, 2011 3:30 am
Forum: Critical Reasoning
Topic: Brain
Replies: 2
Views: 939

I still don't understand why E is correct here. though ur image is not completely visible... I can still guess the correct answer... the question is an inference based question... anything outside of what is written in the stimulus cannot be inferred... anything to do with the loudness of sound or ...

by neerajkumar1_1

Tue Aug 02, 2011 3:25 am
Forum: Critical Reasoning
Topic: Brain
Replies: 2
Views: 939

hi, follow the link... http://www.beatthegmat.com/at-least-one-solution-t66137.html Also logically option A is the only one which has no solution imagine can - 2^n = (-1/2)^n ??? it will never be equal to each other... best case... try putting n=0 u will get -1 = 1 otherwise though with odd powers o...

by neerajkumar1_1

Sat Apr 23, 2011 11:17 pm
Forum: Problem Solving
Topic: MGCAT....700 - 800Q
Replies: 4
Views: 1856